LSAT and Law School Admissions Forum

Get expert LSAT preparation and law school admissions advice from PowerScore Test Preparation.

 Administrator
PowerScore Staff
  • PowerScore Staff
  • Posts: 8950
  • Joined: Feb 02, 2011
|
#26722
This is a Strengthen question, where we're asked to find an answer choice that supports the journalist's argument. So let's start with the argument and see what the journalist believes.

The argument here is straightforward enough: from the observation that people with iron-rich diets are much more prone to develop Parkinson's disease than those whose diets contain less iron, the journalist concludes that limiting the consumption of food with a lot of iron (meat, seafood, etc) should reduce the chance of developing Parkinson's.

This is a causal argument--we move from the correlation of iron-rich diets and a high incidence of Parkinson's to the conclusion that iron is a cause of Parkinson's and thus limiting this cause should reduce its effect--and it seems, at least superficially, to make sense. But causal arguments on the LSAT are pretty much always dubious affairs. There's simply a lot that can go wrong when you attempt to tie two events together in an exclusive, active way, as LSAT authors do when endorsing causal relationships. This is why you see causality in so many Strengthen and Weaken questions: its suspect nature makes it ripe for manipulation.

Generally the most effective way to weaken causality (although hardly the only way) is to introduce an alternate cause. Doing so immediately disrupts the presumed relationship and crushes the author's conclusion. With Strengthen questions you very often see the opposite: you can bolster a causal claim by simply removing a potential competing cause. It's not the most powerful support in the world, but it does help: saying that an alternative cause isn't the reason for the observed effect means the author's claimed cause is more likely to be. This is a crucial idea, as it not only happens with some frequency, but it's also at the heart of some of the toughest Strengthen questions: it's just a counterintuitive notion on the surface, that removing/negating something that hasn't been discussed in the stimulus could in fact lend more credibility to the idea in question.

So how does that work here? Well, consider some alternate causes to dietary iron that could lead to a higher incidence of Parkinson's disease, and still fit with the facts as given. What if, for instance, smoking cigarettes is really the reason people get Parkinson's, and nicotine also makes you crave steak/seafood/iron-rich meals? That would clearly weaken this conclusion ("smoking has long been known as both the leading cause of Parkinson's and a trigger for iron consumption" or whatever). So how would you strengthen against that idea? Negate it: "smoking is a well-known Parkinson's contributor and smokers are notorious for diets low in iron." Seems irrelevant at first, sure, but when you think about it you've just removed a competing theory, thereby making the theory on offer more compelling.

That's a demonstration of the principle at work, of course, and not specific to the answers in this question, but hopefully it serves to show the process by which eliminating an alt. cause can, in a rather roundabout way, offer support.

What happens in the correct answer choice in #21 follows the same mechanism. In (A) we're told that people genetically prone to develop Parkinson's don't eat more iron than people without a genetic predisposition. In other words, a genetic predisposition to getting the disease--something that clearly is a cause--isn't related to the high-iron correlation noted in the stimulus, thus making iron, and not genetics, a more likely contributor. We've removed genetics from the equation and, in so doing, made iron's role more prominent. And thus the author's conclusion about iron --> Parkinson's more likely.

To test this, imagine answer choice (A) stated as its opposite: people with a genetic predisposition to Parkinson's consume MUCH more iron than those without those genes. What would that do here? It would destroy the author's view! If a genetic predisposition, something that undeniably makes you more likely to get the disease, also makes you eat more iron (or if the two are for whatever reason associated), then reducing iron is irrelevant--it's your genes, not your diet--and the conclusion can be dismissed. So by saying genes and iron consumption aren't related, that iron consumption is the same regardless of genetics, we're taking genetics out of it and making iron a more attractive cause.

And that's why (A) strengthens.

I won't go through the rest of the answers since I imagine the difficulty here is understanding why (A) is right rather than why B-E are wrong. Tricky concept, but one well-worth knowing.

Good luck :)
 maximbasu
  • Posts: 59
  • Joined: May 19, 2016
|
#27961
Hi,
Thanks for the reply (I assume this is Dave Killoran; if not, thank you anyway).

Why is D wrong?
I reasoned that since iron in some foods is less easily absorbed by the body than others, then food such as meats and seafoods must be foods who's Iron is easily absorbed by the body and thus you should stay away from these foods.

Is that stating more than in the stimulus?

MB
 Jon Denning
PowerScore Staff
  • PowerScore Staff
  • Posts: 907
  • Joined: Apr 11, 2011
|
#28066
Hi MB - thanks for the question!

(D) requires, as you surmise, an awfully lot of assuming on the part of the reader. That is, without it telling us which foods' iron is more/less easily absorbed, there's absolutely no way we can know that it has any effect whatsoever on the argument in the stimulus.

Consider: if the iron in iron-rich foods like meat and seafood is easily absorbed into the body, then sure, cutting those out would go a long way towards reducing overall iron levels, and if iron really is a cause for Parkinson's then the suggestion in the conclusion seems more reasonable.

Alternatively though what if the iron in iron-rich foods is much less easily absorbed than iron in lower-level-iron food? Would cutting out meat and seafood really help lower levels? Perhaps not. In which case the suggestion in the conclusion makes little sense, and (D) is something of a harmful idea.

The inability to know is all it takes to get rid of (D). (As is the fact that this is a causal argument where we're trying to strengthen the iron :arrow: Parkinson's connection, and (D) does absolutely nothing to that relationship)

This is a common trap where the test makers are hoping you'll make assumptions or add your own information to a wrong answer choice. Don't do it! Answers will be right or wrong on their own, without any help from you :)
 mokkyukkyu
  • Posts: 97
  • Joined: Aug 17, 2016
|
#28313
So the reason A is correct is because its eliminating "predisposition" as the cause...is this what you are saying?
 Nikki Siclunov
PowerScore Staff
  • PowerScore Staff
  • Posts: 1362
  • Joined: Aug 02, 2011
|
#28566
This is largely correct.

Imagine what would happen if answer choice (A) were not true: what if people who are predisposed to Parkinson's actually had higher levels of iron in their blood? In that case, lowering the amount of iron one consumes would probably not reduce their chances of getting the disease - they are predisposed to it already. The opposite of answer choice (A) would make it unclear whether it's iron or genetics that play a critical role in the development of Parkinson's.

But, what if answer choice (A) were true? Assuming that the premises are also true, we'd have the following critical pieces of information:
  • 1. The amount of iron in one's blood correlates with an increased risk of Parkinson's disease.
    2. The amount of iron in one's blood does not correlate with one's genetic predisposition to the disease.
If both of these are true, then the genetic predisposition to the disease, if such a predisposition even exists, cannot explain the increased amount of iron among those who develop Parkinson's. This strengthens the hypothesis that iron and Parkinson's are causally linked.

Hope this clears it up!

Thanks,
 15veries
  • Posts: 113
  • Joined: Sep 25, 2016
|
#30913
I chose A in the first round but now I don't know...
I feel like A kind of weaken the argument for some reason...
So if they have the same standards (eat the same amount of iron) genetic predisposition seems the cause because that's their difference...
Where did I go wrong?
 lilyzhang
  • Posts: 2
  • Joined: Oct 17, 2016
|
#30923
Hello!

When I was reviewing this question, I thought A was correct because it removes the possibility of a reversed causal relationship. In other words, I believe A makes it that if one is predisposed to Parkinson's Disease, this does not make the person more likely to have a high iron diet, thus strengthening the argument.

Is this reasoning correct?

Thanks!
User avatar
 Jonathan Evans
PowerScore Staff
  • PowerScore Staff
  • Posts: 727
  • Joined: Jun 09, 2016
|
#31005
Hi, 15veries and Lilyzhang,

Good questions. Lily, you've actually done a good job with the reasoning, and you will note that your reasoning matches with that of the explanation at the top of this post. This is a defender answer choice that reduces the possibility that it's actually the genetic predisposition to Parkinson's, correlated with high iron levels, that causes the increased incidence of Parkinson's in people with higher than average iron levels. It does indeed eliminate an alternate cause, and thus strengthens the argument. 15veries, good job on your first pass. I hope this explanation along with that of the Administrator above can clear up this problem for you!
 jw190
  • Posts: 11
  • Joined: Dec 07, 2016
|
#32148
Fantastic explanation! I initially got this one right, but more because of elimination than actually knowing why A was correct. This really cleared it up for me! Thanks Administrator! :lol:
 mN2mmvf
  • Posts: 113
  • Joined: Jul 06, 2017
|
#38408
I'm still struggling a bit with this explanation: "In other words, a genetic predisposition to getting the disease--something that clearly is a cause--isn't related to the high-iron correlation noted in the stimulus, thus making iron, and not genetics, a more likely contributor. We've removed genetics from the equation and, in so doing, made iron's role more prominent. And thus the author's conclusion about iron --> Parkinson's more likely."

If we remove iron from the equation, doesn't that remove an alternate cause and make it more likely genetics is the cause after all?

Get the most out of your LSAT Prep Plus subscription.

Analyze and track your performance with our Testing and Analytics Package.